Hi lsatquestions!
Happy to address why answer choice (B) is incorrect.
The description you provided mentioned one flaw in the stimulus--that the author sides with one side over the other. In general, that's an accurate description, but it can be more specific in terms of unpacking why siding with one over the other is a
flaw. Namely, it might be a flaw to choose one side over the other if there is no reason to choose one over the other, or if there are better reasons for choosing the option that one did not choose.
This seems to be what is going on in the stimulus. It is saying,
(S) AND (H) GA
That is, if S and H both support the proposal, then it will have government approval. And we're given
GA, which is in the contrapositive of the above diagram:
GA (S) OR (H)
That is, it did not have government approval, which means that it was missing either S's or H's support. The author concludes that H must not have supported it. This is a flaw, because, without more information, we don't know why it couldn't have been S who didn't support it. What is more, it is a conclusion reached "despite [H's] claims to the contrary." Given that extra piece, if anything it'd be more reasonable to conclude that S didn't support it.
This flaw is paralleled in answer choice (A). In that answer choice, there are competing accounts of an accident from Morgan's newspaper report and from the TV news. The author then concludes in favor of one/against the other, but without any additional information that warrants that choice. Additionally, we have something similar to H's claims to the contrary--"Morgan witnessed the accident from his kitchen window." There's no claim that the TV news witnessed it, so if anything, it'd be more reasonable to conclude that Morgan was correct.
Answer choice (B) also involves choosing between two options, but they aren't
incompatible options. Both the private institute and the city government can be right about their claims. This aspect of answer choice (B) thus doesn't parallel the flaw in the stimulus. There's additionally no clear reason to prefer either the private institute or the city government. Thus it also doesn't parallel the H's "claims to the contrary" portion of the stimulus, which makes it more reasonable to choose one option over the other.